LSAT 144 – Section 2 – Question 17
You need a full course to see this video. Enroll now and get started in less than a minute.
Target time: 1:24
This is question data from the 7Sage LSAT Scorer. You can score your LSATs, track your results, and analyze your performance with pretty charts and vital statistics - all with a Free Account ← sign up in less than 10 seconds
Question QuickView |
Type | Tags | Answer Choices |
Curve | Question Difficulty |
Psg/Game/S Difficulty |
Explanation |
---|---|---|---|---|---|---|---|
PT144 S2 Q17 |
+LR
| Most strongly supported +MSS Net Effect +NetEff | A
13%
157
B
6%
162
C
10%
159
D
71%
166
E
0%
151
|
146 155 165 |
+Harder | 148.975 +SubsectionMedium |
City leader: If our city adopts the new tourism plan, the amount of money that tourists spend here annually will increase by at least $2 billion, creating as many jobs as a new automobile manufacturing plant would. It would be reasonable for the city to spend the amount of money necessary to convince an automobile manufacturer to build a plant here, but adopting the tourism plan would cost less.
Summary
City leader: If we adopt the new tourism plan, tourists will spend at least $2 billion more each year here, creating as many jobs as a new car manufacturing plant would. It would be reasonable to spend money to attract a car manufacturing plant, but the tourism plan would cost less.
When determining the reasonableness of implementing something that would create job growth for the city, cost is an important factor.
Adopting the new tourism plan would be reasonable.
Adopting the new tourism plan would be economically beneficial for the city.
A
The city should implement the least expensive job creation measures available.
Unsupported. We know that the new tourism plan is cheaper than attracting a car manufacturing plant, but we do not know that it is the least expensive job creating measure available. The stimulus doesn’t discuss the least expensive measures or whether they should be implemented.
B
In general, it is reasonable for the city to spend money to try to convince manufacturing companies to build plants in the city.
Unsupported. The stimulus tells us that it would be reasonable for the city to spend the money necessary to convince an automobile manufacturer to build a plant, but it does not discuss the reasonableness of attracting manufacturing companies in general.
C
The city cannot afford both to spend money to convince an automobile manufacturer to build a plant in the city and to adopt the new tourism plan.
Unsupported. We are not told how much money the city has or whether it can afford to attract a car manufacturer and implement the new tourism plan. We simply don’t know.
D
It would be reasonable for the city to adopt the new tourism plan.
Strongly supported. The new tourism plan would create as many jobs as a new car factory. It would be reasonable for the city to spend the money to attract the car factory. The new tourism plan would cost less. So it would be reasonable for the city to adopt the new tourism plan.
E
The only way the city can create jobs is by increasing tourism.
Anti-supported. The stimulus explicitly states that a new car manufacturing plant would create as many jobs as increased tourism. So increasing tourism is not the only way that the city can create jobs.
Take PrepTest
Review Results
LSAT PrepTest 144 Explanations
Section 1 - Reading Comprehension
- Passage 1 – Passage
- Passage 1 – Questions
- Passage 2 – Passage
- Passage 2 – Questions
- Passage 3 – Passage
- Passage 3 – Questions
- Passage 4 – Passage
- Passage 4 – Questions
Section 2 - Logical Reasoning
- Question 01
- Question 02
- Question 03
- Question 04
- Question 05
- Question 06
- Question 07
- Question 08
- Question 09
- Question 10
- Question 11
- Question 12
- Question 13
- Question 14
- Question 15
- Question 16
- Question 17
- Question 18
- Question 19
- Question 20
- Question 21
- Question 22
- Question 23
- Question 24
- Question 25
- Question 26
Section 3 - Logical Reasoning
- Question 01
- Question 02
- Question 03
- Question 04
- Question 05
- Question 06
- Question 07
- Question 08
- Question 09
- Question 10
- Question 11
- Question 12
- Question 13
- Question 14
- Question 15
- Question 16
- Question 17
- Question 18
- Question 19
- Question 20
- Question 21
- Question 22
- Question 23
- Question 24
- Question 25
Leave a Reply
You must be logged in to post a comment. You can get a free account here.